10
$\begingroup$

We have $N$ students and $n$ exams. We need to select $n$ out of the students using the grade of those exams. The procedure is as follows:

1- We set some ordering on the exams.

2- Going through this order, In every exam, the best student will be selected and we forget about his/her grade in the next exams.

Now we have all grades and we know that before setting the ordering on the exams, all the student are hopeful to be selected. How large can $N$ be in terms of $n$?

(As far as I remember, we proved a $2n+c$ lower bound and an $O(n\log n)$ upper bound for $N$.)

$\endgroup$
7
  • 3
    $\begingroup$ Does "all the student are hopeful" mean that, for every student, there exists an ordering of the exams under which that student will be among the $n$ selected? $\endgroup$ Apr 11, 2019 at 13:57
  • $\begingroup$ Yes, exactly... $\endgroup$
    – Morteza
    Apr 11, 2019 at 14:46
  • 1
    $\begingroup$ An easy way to get $2n-1$ as a lower bound: Fix $n-1$ students that are the best in every one of the $n$ exams (i.e., these $n-1$ students occupy the top $n-1$ spots for each exam, in some (irrelevant) order). Then let there be $n$ more students, each one in the $n^{\mathrm{th}}$ spot on exactly one exam. The first $n-1$ students will be selected for any ordering of the exams, and one of the other $n$ students will be selected if and only if his/her exam is held last. $\endgroup$
    – Will Brian
    Apr 11, 2019 at 15:04
  • 2
    $\begingroup$ Could the title be related to the mathematical contents of the question? right now it seems to be an appeal for closing votes... $\endgroup$
    – YCor
    Apr 11, 2019 at 19:19
  • 2
    $\begingroup$ What is the constant at $n\log_2 n$ in your upper bound? Is there still a gap? $\endgroup$ Apr 12, 2019 at 9:12

2 Answers 2

3
$\begingroup$

My initial argument was erroneous. I'm rewriting everything completely.

Denote by $f(n)$ the maximal value of $N$ for that value of $n$. I claim that $$ f(k+\ell)\geq f(k)+f(\ell)+k \qquad\text{for all $k\leq \ell$.} \qquad\qquad(*) $$ In a standard way, this shows that $f(n)\geq 1+\frac12n\log_2 n$.

To prove $(*)$, assume that we have $k+\ell$ exams. Take a set $X$ of $f(k)$ students hopeful for the first $k$ exams and hopeless for the last $\ell$, and take a set $Y$ of $f(\ell)$ students performing in the opposite way. Finally, take a set $Z$ of $k$ students, each of which wins his own exam in the first group and his own exam in the last group and not attenfing the others.

Now, if the first $k$ exams are taken into account first, then $Z$ win their exams and become out of competition, so $Y$ are still hopeful. Similarly, $X$ stay hopeful if the last $\ell$ exams are taken into account first.

$\endgroup$
3
  • $\begingroup$ Perhaps, now this is almost the same as @WillBrian's construction... $\endgroup$ Apr 12, 2019 at 9:13
  • $\begingroup$ Yes, this is basically the same idea. I like the more general way you've expressed it, though, and of course it's nice that you've gotten a better constant for your $n \log n$ term. +1 $\endgroup$
    – Will Brian
    Apr 12, 2019 at 13:31
  • $\begingroup$ The idea was very nice to me. $\endgroup$
    – Morteza
    Apr 12, 2019 at 19:22
3
$\begingroup$

A better-than-linear lower bound for $N$ is $\frac{1}{4} (n \log_2 n + n)$.

To see why, let's look at what you can do when $n$ is a power of $2$.

For $n = 2^1 = 2$, we may take $N_2=3$. Our $N=3$ students are called $a,b,c$, and in our $n=2$ exams, the top two students are:

Exam 1: $\ a \ \ b$

Exam 2: $\ a \ \ c$

It is clear that student $a$ will be selected in the first round, and then either $b$ or $c$ will be selected next, depending on the ordering of the exams.

For $n = 2^2 = 4$, we may take $N_4=8$. Our $N=8$ students are called $a,b,c,d,e,f,g,h$, and in our $n=4$ exams, the top four students are:

Exam 1: $\ a \ \ b \ \ c \ \ e$

Exam 2: $\ a \ \ b \ \ c \ \ f$

Exam 3: $\ a \ \ b \ \ d \ \ g$

Exam 4: $\ a \ \ b \ \ d \ \ h$

It is clear that student $a$ and $b$ will be selected in the first two rounds. No one else is guaranteed selection, but it is not difficult to check that any given student is selected for some ordering of the four exams.

For $n = 2^3 = 8$, we may take $N_8=20$. Our $N=20$ students are called $a,b,c,d,e,f,g,h,i,j,k,l,m,n,o,p,q,r,s,t$, and in our $n=8$ exams, the top eight students are:

Exam 1: $\ a \ \ b \ \ c \ \ d \ \ \ e \ \ g \ \ i \ \ m$

Exam 2: $\ a \ \ b \ \ c \ \ d \ \ \ e \ \ g \ \ i \ \ n$

Exam 3: $\ a \ \ b \ \ c \ \ d \ \ \ e \ \ g \ \ j \ \ o$

Exam 4: $\ a \ \ b \ \ c \ \ d \ \ \ e \ \ g \ \ j \ \ p$

Exam 5: $\ a \ \ b \ \ c \ \ d \ \ f \ \ h \ \ k \ \ q$

Exam 6: $\ a \ \ b \ \ c \ \ d \ \ f \ \ h \ \ k \ \ r$

Exam 7: $\ a \ \ b \ \ c \ \ d \ \ f \ \ h \ \ l \ \ s$

Exam 8: $\ a \ \ b \ \ c \ \ d \ \ f \ \ h \ \ l \ \ t$

This time students $a - d$ are guaranteed selection, but no one else is. Again, it is not difficult to check that any given student is selected for some ordering of the four exams.

Without going to the trouble of writing down a formal recurrence for this pattern, you can check that if $n = 2^k$ then a solution like this leads to a situation where $$N = 2^{k-1} \cdot 1 + 2^{k-2} \cdot 2 + 2^{k-3} \cdot 4 + 2 \cdot 2^{k-2} + 1 \cdot 2^{k-1} + 2^k = 2^{k-1}(k + 2).$$ Putting everything in terms of $n$, this is $N_{2^k} = \frac{n}{2}(\log_2 n + 2)$.

For some $n$ that is not a power of $2$, we can (crudely) just round down to the nearest power of $2$ for the desired solution: pick $k$ such that $2^k < n \leq 2^{k+1}$ and then observe that $$N_n \geq N_{2^k} \geq (k+2)2^{k-1} = \frac{1}{4}(2^{k+1} \cdot k + 2 \cdot 2^{k+1}) = \frac{1}{4}(2^{k+1}(k+1) + 2^{m+1}) \geq \frac{1}{4}(n \log_2 n+n)$$ as claimed.

$\endgroup$
1
  • $\begingroup$ Thank you for the idea. $\endgroup$
    – Morteza
    Apr 13, 2019 at 15:11

Your Answer

By clicking “Post Your Answer”, you agree to our terms of service and acknowledge that you have read and understand our privacy policy and code of conduct.

Not the answer you're looking for? Browse other questions tagged or ask your own question.